Creating a Confidence Interval for θ

Click For Summary
SUMMARY

The discussion focuses on deriving a confidence interval for the parameter θ using the asymptotic normality of the estimator \(\widehat{\theta}\). The key formula presented is \(\left( \widehat{\theta} \pm u_{1-\frac{\alpha}{2}}\frac{1}{\sqrt{nF\left(\widehat{\theta}\right)}}\right)\), where \(u_{1-\frac{\alpha}{2}}\) represents the quantile of the standard normal distribution. The relationship between the standard normal distribution and the probability of the inequality \(-u < \sqrt{n F(\theta)} \left(\hat \theta - \theta\right) < u\) is also established, emphasizing the importance of understanding this inequality in constructing the confidence interval.

PREREQUISITES
  • Understanding of asymptotic normality in statistics
  • Familiarity with confidence intervals and their construction
  • Knowledge of standard normal distribution and quantiles
  • Basic proficiency in statistical notation and expressions
NEXT STEPS
  • Study the derivation of confidence intervals in statistical inference
  • Learn about the properties of the standard normal distribution
  • Explore the concept of asymptotic distributions in statistics
  • Investigate the implications of the Central Limit Theorem on confidence intervals
USEFUL FOR

Statisticians, data analysts, and researchers involved in statistical modeling and inference, particularly those interested in constructing and interpreting confidence intervals for parameters.

twoflower
Messages
363
Reaction score
0
Let's say we know this:

<br /> \sqrt{n}\left(\widehat{\theta} - \theta\right) \sim \mathcal{N}\left(0, \frac{1}{F(\theta)}\right)<br />

How do we get from this information to this expression of confidence interval for \theta?

<br /> \left( \widehat{\theta} \pm u_{1-\frac{\alpha}{2}}\frac{1}{\sqrt{nF\left(\widehat{\theta}\right)}}\right)<br />

Where u_{1-\frac{\alpha}{2}} is appropriate quantil of standard normal distribution.

Thank you.
 
Physics news on Phys.org
If a is the value from Z (standard normal) with area {\alpha}/2 to its right, you know the value of

<br /> \Pr\left(-u &lt; \sqrt{n F(\theta)} \left(\hat \theta - \theta\right) &lt; u) <br />

because of your stated approximate normality result. That means the event

<br /> -u &lt; \sqrt{n F(\theta)} \left(\hat \theta - \theta\right) &lt; u<br />

has a known probability of occurring. What can you do with this inequality? (Try some work and include it with your next question if you are unsure.)
 
Question: A clock's minute hand has length 4 and its hour hand has length 3. What is the distance between the tips at the moment when it is increasing most rapidly?(Putnam Exam Question) Answer: Making assumption that both the hands moves at constant angular velocities, the answer is ## \sqrt{7} .## But don't you think this assumption is somewhat doubtful and wrong?

Similar threads

  • · Replies 2 ·
Replies
2
Views
2K
  • · Replies 1 ·
Replies
1
Views
1K
  • · Replies 22 ·
Replies
22
Views
973
  • · Replies 2 ·
Replies
2
Views
1K
  • · Replies 11 ·
Replies
11
Views
2K
Replies
7
Views
2K
  • · Replies 4 ·
Replies
4
Views
1K
Replies
4
Views
2K
  • · Replies 3 ·
Replies
3
Views
2K
  • · Replies 3 ·
Replies
3
Views
1K